LSAT and Law School Admissions Forum

Get expert LSAT preparation and law school admissions advice from PowerScore Test Preparation.

User avatar
 Dave Killoran
PowerScore Staff
  • PowerScore Staff
  • Posts: 5852
  • Joined: Mar 25, 2011
|
#42384
Complete Question Explanation
(The complete setup for this game can be found here: lsat/viewtopic.php?t=16021)

The correct answer choice is (E)

This is the first question to address the “speech” subgroup established in the initial conditions of the game. Since M, P, and Q form the speech subgroup, and M cannot be seated at table 3, it follows that the one sponsor seated at table 3 who gives a speech must be either P or Q. Additionally, since at least two sponsors from the group P, Q, and Z must be seated at table 3, we can infer that Z must be seated at table 3:
J97_Game_#1_#5_diagram 1.png
Thus, answer choice (E) is correct.
You do not have the required permissions to view the files attached to this post.

Get the most out of your LSAT Prep Plus subscription.

Analyze and track your performance with our Testing and Analytics Package.